Adjust My Previous Question
I guess, the logic structure of the stimulus is like the following: "Only the fact S bacilli are...
Batman on September 11, 2014
  • June 1997 LSAT
  • SEC4
  • Q20
2
Replies
Help
Please explain this. Why (d) is not an answer? Thanks,
Batman on September 9, 2014
  • June 1997 LSAT
  • SEC4
  • Q18
2
Replies
Help
Please explain what logical structure in the stimulus is parallel with the choice (b)'s? Thanks,
Batman on September 9, 2014
  • June 1997 LSAT
  • SEC4
  • Q20
2
Replies
D And E
Could you pleas explain the difference between answer choices D and E?
Derek on September 5, 2014
  • December 2009 LSAT
  • SEC3
  • Q7
1
Reply
Why A?
If the conclusion is that the doctors claim they can produce a vaccine that will produce permanen...
Derek on September 3, 2014
  • June 2009 LSAT
  • SEC3
  • Q11
2
Replies
I Am Lost
For some reason I am puzzled by this question and have spent too much time staring at the words. ...
Derek on September 3, 2014
  • June 2009 LSAT
  • SEC3
  • Q9
2
Replies
question
Can you please explain why A is not correct? The original argument compares two different compani...
NicoCapri on September 2, 2014
  • June 2004 LSAT
  • SEC2
  • Q11
1
Reply
Help
Is this method of reasoning question or evaluation question? What does the answer mean? Thanks
Batman on August 29, 2014
  • June 1996 LSAT
  • SEC4
  • Q5
2
Replies
Help
I chose (e) as the right answer. However, I don't vividly get understood what difference between ...
Batman on August 28, 2014
  • December 1996 LSAT
  • SEC2
  • Q16
2
Replies
Need your help
Could you please show me how draw diagram of the reasoning method on the stimulus? I don't think ...
Batman on August 25, 2014
  • June 2004 LSAT
  • SEC3
  • Q25
2
Replies
Please Explain
Why is "B" correct?
KDA86 on August 23, 2014
  • October 2012 LSAT
  • SEC1
  • Q3
1
Reply
Please Explain
How would you set this game up?
KDA86 on August 21, 2014
  • October 2012 LSAT
  • SEC2
  • Q5
1
Reply
Rule #3
Can you explain the rule #3 "At least as many French novels....." I believe my misinterpretatio...
MGN2014 on August 18, 2014
  • October 2000 LSAT
  • SEC3
  • Q8
2
Replies
Difference between A and E?
I'm confused how would it be E instead of A.
MGN2014 on August 16, 2014
  • October 2000 LSAT
  • SEC3
  • Q15
1
Reply
At least terminology
I keep getting tripped up with the "at least" phrase that throws me off. Because I selected C bec...
MGN2014 on August 16, 2014
  • October 2000 LSAT
  • SEC3
  • Q14
3
Replies
Help
I could hardly distinguish the difference between (d) and (e). Why is (d) the answer to this ques...
Batman on August 15, 2014
  • June 1997 LSAT
  • SEC1
  • Q13
2
Replies
Help
Could you please explain the reason why (E) shouldn't be the answer? Thanks,
Batman on August 14, 2014
  • June 1997 LSAT
  • SEC1
  • Q8
2
Replies
Setup
Is it possible to view the set up of this game?
MGN2014 on August 14, 2014
  • October 2000 LSAT
  • SEC3
  • Q3
1
Reply
Help
How could both approach be complement each other according to the passage? I don't understand tha...
Batman on August 14, 2014
  • June 1997 LSAT
  • SEC1
  • Q2
2
Replies
Why?
I understand on a sentence level why this sentence could follow. But I am still confused as to wh...
Katie10 on August 13, 2014
  • December 2009 LSAT
  • SEC5
  • Q26
1
Reply